www.vorhilfe.de
Vorhilfe

Kostenlose Kommunikationsplattform für gegenseitige Hilfestellungen.
Hallo Gast!einloggen | registrieren ]
Startseite · Forum · Wissen · Kurse · Mitglieder · Team · Impressum
Forenbaum
^ Forenbaum
Status Englisch
  Status Grammatik
  Status Lektüre
  Status Korrekturlesen
  Status Übersetzung
  Status Sonstiges (Englisch)

Gezeigt werden alle Foren bis zur Tiefe 2

Navigation
 Startseite...
 Neuerdings beta neu
 Forum...
 vorwissen...
 vorkurse...
 Werkzeuge...
 Nachhilfevermittlung beta...
 Online-Spiele beta
 Suchen
 Verein...
 Impressum
Das Projekt
Server und Internetanbindung werden durch Spenden finanziert.
Organisiert wird das Projekt von unserem Koordinatorenteam.
Hunderte Mitglieder helfen ehrenamtlich in unseren moderierten Foren.
Anbieter der Seite ist der gemeinnützige Verein "Vorhilfe.de e.V.".
Partnerseiten
Weitere Fächer:

Open Source FunktionenplotterFunkyPlot: Kostenloser und quelloffener Funktionenplotter für Linux und andere Betriebssysteme
Forum "Folgen und Reihen" - Grenzwert
Grenzwert < Folgen und Reihen < eindimensional < reell < Analysis < Hochschule < Mathe < Vorhilfe
Ansicht: [ geschachtelt ] | ^ Forum "Folgen und Reihen"  | ^^ Alle Foren  | ^ Forenbaum  | Materialien

Grenzwert: Frage (beantwortet)
Status: (Frage) beantwortet Status 
Datum: 14:48 Fr 14.03.2008
Autor: domenigge135

Hallo. Ich habe ein kleines Problem zu 2 Aufgaben:

Bestimmen Sie, falls vorhanden den Grenzwert:
[mm] a)\limes_{x\rightarrow\infty}\bruch{\wurzel{x^2+1}}{x+1} [/mm]
[mm] b)\limes_{x\rightarrow 0}\bruch{\wurzel{x}}{|x|} [/mm]

Ich habe von oben bis unten alles Methoden probiert. Auch die von L'hospital. Aber keine bringt mich auf die richtige Lösung. Könntet ihr mir bitte helfen??? Wäre sehr dankbar.


        
Bezug
Grenzwert: Tipps
Status: (Antwort) fertig Status 
Datum: 14:52 Fr 14.03.2008
Autor: Loddar

Hallo domenigge!


Bei der ersten Aufgabe mal in Zähler und Nenner jeweils $x_$ ausklammern und kürzen.


Bei der 2. Aufgabe kannst Du zunächst die Betragsstriche weglassen, da [mm] $\wurzel{x}$ [/mm] sowieso nur für positive x definiert ist. Anschließend den Term mittels MBPotenzgesetz zusammenfassen.


Gruß
Loddar


Bezug
                
Bezug
Grenzwert: Frage (beantwortet)
Status: (Frage) beantwortet Status 
Datum: 15:00 Fr 14.03.2008
Autor: domenigge135

a) Das mit den Ausklammern ist ja klar. Aber wie sieht das dann für den Zähler aus? Wegen der Klammer! :-(

b) [mm] \bruch{x}{|x|}=\bruch{x}{\wurzel{x^2}}=\bruch{x}{x} [/mm] Da x nun gegen 0 geht, müsste dort stehen [mm] \bruch{0}{0} [/mm] Kann ich nun L'hospital anwenden??? Also: [mm] \bruch{1}{2x}??? [/mm] Denn das ist ja auch nicht definiert!!!

Bezug
                        
Bezug
Grenzwert: Antwort
Status: (Antwort) fertig Status 
Datum: 15:07 Fr 14.03.2008
Autor: schachuzipus

Hallo domenigge,

> a) Das mit den Ausklammern ist ja klar. Aber wie sieht das
> dann für den Zähler aus? Wegen der Klammer! :-(

klammere zunächst unter der Wurzel [mm] $x^2$ [/mm] aus und zeihe es dann raus:

[mm] $\sqrt{x^2+1}=\sqrt{x^2\cdot{}\left(1+\frac{1}{x^2}\right)}=x\cdot{}\sqrt{...}$ [/mm]

>  
> b) [mm]\bruch{x}{|x|}=\bruch{x}{\wurzel{x^2}}=\bruch{x}{x}[/mm]

Wo ist denn die Wurzel aus dem Zähler hin?

[mm] $\lim\limits_{x\to 0}\frac{\sqrt{x}}{|x|}=\lim\limits_{x\to 0}\frac{\sqrt{x}}{x}=\lim\limits_{x\to 0}\frac{x^{\frac{1}{2}}}{x^1}=...$ [/mm]

> Da x nun gegen 0 geht, müsste dort stehen [mm]\bruch{0}{0}[/mm] Kann ich
> nun L'hospital anwenden??? Also: [mm]\bruch{1}{2x}???[/mm] Denn das
> ist ja auch nicht definiert!!!


LG

schachuzipus

Bezug
                                
Bezug
Grenzwert: Frage (beantwortet)
Status: (Frage) beantwortet Status 
Datum: 15:10 Fr 14.03.2008
Autor: domenigge135

Oh Sorry... Das war dann wohl mein Fehler. Da steht: [mm] \limes_{x\rightarrow\0}\bruch{x}{|x|} [/mm] und nicht das, was ich zu Anfang geschrieben habe!!!

Bezug
                                        
Bezug
Grenzwert: Antwort
Status: (Antwort) fertig Status 
Datum: 15:21 Fr 14.03.2008
Autor: Marcel

Hallo Domenigge,

> Oh Sorry... Das war dann wohl mein Fehler. Da steht:
> [mm]\limes_{x\rightarrow 0}\bruch{x}{|x|}[/mm] und nicht das, was
> ich zu Anfang geschrieben habe!!!

1. Fall: Es sei $x > 0$.
Was ist dann [mm] $\frac{x}{|x|}$? [/mm]

2. Fall: Es sei $x < 0$:
Was ist dann [mm] $\frac{x}{|x|}$? [/mm]

Beachte:
[mm] $|x|=\begin{cases} x, & \mbox{für } x \ge 0 \\ -x, & \mbox{für } x \le 0 \end{cases}$ [/mm]

Was folgt daraus für

[mm] $\lim_{x > 0 \mbox{ und } x \to 0}\frac{x}{|x|}$? [/mm]

Was folgt daraus für

[mm] $\lim_{x < 0 \mbox{ und } x \to 0}\frac{x}{|x|}$? [/mm]

Folgerung:
Kann [mm] $\lim_{x \to 0} \frac{x}{|x|}$ [/mm] dann existieren?

Zur Kontrolle auch mal ein Schaubild des Graphen der Funktion [mm] $f(x)=\frac{x}{|x|}$ [/mm] ($x [mm] \in \IR \backslash\{0\}$): [/mm]

[Dateianhang nicht öffentlich]

Gruß,
Marcel

Dateianhänge:
Anhang Nr. 1 (Typ: jpg) [nicht öffentlich]
Bezug
                                                
Bezug
Grenzwert: Frage (beantwortet)
Status: (Frage) beantwortet Status 
Datum: 15:25 Fr 14.03.2008
Autor: domenigge135

Gut dann erhalte ich einmal 1 und einmal -1 als Ergebnis. Das hat erstens nichts mit stetigkeit zu tun. Somit kann es dann wohl keinen Grenzwert geben.

Bezug
                                                        
Bezug
Grenzwert: richtig
Status: (Antwort) fertig Status 
Datum: 15:27 Fr 14.03.2008
Autor: Loddar

Hallo domenigge!


Richtig erkannt ...


Gruß
Loddar


Bezug
                                                                
Bezug
Grenzwert: Frage (beantwortet)
Status: (Frage) beantwortet Status 
Datum: 15:31 Fr 14.03.2008
Autor: domenigge135

Na dann bin ich ja erleichtert. Ich habe allerdings noch ein kleines Problem, bei dem ich irgendwie mit diesem Standarttrick oder L'Hospital nicht weiterkam. Und zwar die Aufgabe: [mm] \limes_{x\rightarrow\1}\bruch{1}{1-x}-\bruch{3}{1-x^2} [/mm]

Gibt es hier auch einen Trick???

Bezug
                                                                        
Bezug
Grenzwert: Antwort
Status: (Antwort) fertig Status 
Datum: 15:48 Fr 14.03.2008
Autor: Marcel

Hallo,

> Na dann bin ich ja erleichtert. Ich habe allerdings noch
> ein kleines Problem, bei dem ich irgendwie mit diesem
> Standarttrick oder L'Hospital nicht weiterkam. Und zwar die
> Aufgabe:
> [mm]\limes_{x\rightarrow 1}\bruch{1}{1-x}-\bruch{3}{1-x^2}[/mm]
>  
> Gibt es hier auch einen Trick???

ja (bei dem Limes bitte keinen Backslash vor die Zahl schreiben, sonst erscheint anstatt [mm] $\lim_{x \to 1}$ [/mm] dort [mm] $\lim_{x \to \1}$, [/mm] d.h. die Zahl $1$ unter dem Limes-Symbol ist nicht sichtbar). Der Trick heißt: Terme zusammenfassen ;-)

[mm] $\frac{1}{1-x}-\frac{3}{1-x^2}=\frac{1+x}{1-x^2}-\frac{3}{1-x^2}=\frac{x-2}{1-x^2}$ [/mm]

Für $x [mm] \to [/mm] 1$ (egal, ob $x > 1$, $x < 1$) strebt der Zähler $x-2$ gegen $-1$.

Der Nenner:

Für $x < 1$ und $x [mm] \to [/mm] 1$ ist stets [mm] $1-x^2 [/mm] > 0$ und strebt gegen $0$, d.h.:

[mm] $\lim_{x < 1 \mbox{ und } x \to 1} \frac{x-2}{1-x^2}=-\infty$ [/mm]

(Symbolisch begründe ich das mal so: " [mm] $\frac{-1}{+0}=-\infty$ [/mm] ")

Analog:

Für $x > 1$ und $x [mm] \to [/mm] 1$ ist stets [mm] $1-x^2 [/mm] < 0$ und strebt gegen $0$, d.h.:

[mm] $\lim_{x > 1 \mbox{ und } x \to 1} \frac{x-2}{1-x^2}=\infty$ [/mm]

(Symbolisch begründe ich das mal so: " [mm] $\frac{-1}{-0}=\infty$ [/mm] ")

Frage:
Kann [mm] $\lim_{x \to 1} \bruch{1}{1-x}-\bruch{3}{1-x^2}=\lim_{x \to 1}\frac{x-2}{1-x^2}$ [/mm] existieren?

Gruß,
Marcel

Bezug
Ansicht: [ geschachtelt ] | ^ Forum "Folgen und Reihen"  | ^^ Alle Foren  | ^ Forenbaum  | Materialien


^ Seitenanfang ^
www.englischraum.de
[ Startseite | Forum | Wissen | Kurse | Mitglieder | Team | Impressum ]